1answer.
Ask question
Login Signup
Ask question
All categories
  • English
  • Mathematics
  • Social Studies
  • Business
  • History
  • Health
  • Geography
  • Biology
  • Physics
  • Chemistry
  • Computers and Technology
  • Arts
  • World Languages
  • Spanish
  • French
  • German
  • Advanced Placement (AP)
  • SAT
  • Medicine
  • Law
  • Engineering
slamgirl [31]
3 years ago
7

Solve for x. Enter your answer in the box.

Mathematics
2 answers:
ahrayia [7]3 years ago
7 0

Answer:

x° = 132°.

Step-by-step explanation:

Given: Hexagon with angle 108°,  113°  132°  75°  160°  and x° .

To find: x° .

Solution: We have given that  Hexagon with angle 108°,  113°  132°  75°  160°  and x° .

By the sum of angle Hexagon property : Sum of all angle of hexagon is 720° .

108°+113° +  132° +  75° + 160°  +x°  =720°

 588 + x°  =720

subtracting both side 588

588 -588 + x° = 720°-588

    x° = 132°

Therefore,  x° = 132° .

coldgirl [10]3 years ago
4 0

interior angles of a hexagon add up to 720.

75+160+132+113+108=588

720-558=162

the x is 162

You might be interested in
8. Identify which equation is a graph of a vertical line.
Y_Kistochka [10]

Answer:

10x = 10.

Step-by-step explanation:

The equation will contain the  x variable only.

4 0
3 years ago
Half of the number, n, plus eight?
3241004551 [841]
Your answer would be A.
3 0
3 years ago
Read 2 more answers
May i know the answer for this please
Naddik [55]

Answer:

14, 16

Step-by-step explanation:

The prime factorization of 224 is 2⁵ × 7. To make it a perfect square, we need the bases to have an even exponent, therefore we divide it by 2 × 7 = 14 to get 16.

4 0
4 years ago
4x^2-5x-15=0 how to find the vertex and y intercept
mezya [45]
ax^2+bx+c=0\\\\the\ vertex:\left(\frac{-b}{2a};\frac{-(b^2-4ac)}{4a}\right)\\\\y-intrcept=c\\=======================================\\\\4x^2-5x-15=0\\\\a=4;\ b=-5;\ c=-15\\\\\frac{-b}{2a}=\frac{-(-5)}{2\cdot4}=\frac{5}{8};\ \frac{-(b^2-4ac)}{4a}=\frac{-[(-5)^2-4\cdot4\cdot(-15)]}{4\cdot4}=-\frac{265}{16}\\\\\boxed{the\ vertex:\left(\frac{5}{8};-\frac{265}{16}\right)}\\\\\boxed{y-intercept:-15}
5 0
3 years ago
|x/3|=1<br><br> please help me &lt;3
Anastaziya [24]
The correct answer is -3
6 0
3 years ago
Other questions:
  • 7. Find the circumcenter of triangle ABC.<br><br> (0, -1)<br> (0, -3)<br> (-4, -1)<br> (-4, -3)
    12·1 answer
  • Kylie knows that she will have to help pay for college. She has a part time job and is able to save $75 every 2 weeks. What is t
    7·2 answers
  • Please help me out.................
    14·1 answer
  • What is the y-intercept of f^-1(x)?
    14·1 answer
  • Factor each completely.<br> b^2 + 8b + 15
    6·1 answer
  • Debby deposits 25% if the money that she earns each week in a savibgs account if debby earns $72 each week, how much money will
    9·1 answer
  • Write and solve a proportion to complete the statement. 12.6 kg= ? ib
    8·1 answer
  • Write 7x7 in exponential form
    15·1 answer
  • A 2 kg hammer is used to nail a 0.002 kg nail with a force of 10 N. How much force is applied to the hammer?
    5·1 answer
  • The least common multiply of 2, 5, 6 and 9 is
    6·2 answers
Add answer
Login
Not registered? Fast signup
Signup
Login Signup
Ask question!